2
$\begingroup$

Let $V$ be an $n$-dimensional vector space over the finite field of cardinality $q$ and let $W_1,\ldots,W_m$ be hyperplanes of $V$ such that $$V=\bigcup_{i=1}^mW_i \,\,\hbox{ and }\,\,0=\bigcap_{i=1}^mW_i.$$

I am interested in a lower bound for $m$ in terms of $n$ and $q$. For instance, when $n=2$, we have $m=q+1$.

The collection $\{W_1,\ldots,W_m\}$ of hyperplanes with the conditions above seems to be the "dual" of a blocking set (for projective plane). A blocking set $S$ in a projective plane is a subset of points with the property that every line intersect non-trivially $S$, and $S$ does not contain a line. So, possibly my question has already an answer in the literature. But I was not able to find it, besides for planes.

$\endgroup$

2 Answers 2

1
$\begingroup$

The trivial bound $m=q+1$ mentioned in the case where $n=2$ is actually true for all $n$ by simple counting: a set of $k$ hyperplanes contains at most $kq^{n-1}+q^{n-2}+\ldots+1$ points, since the "first" hyperplane has $q^{n-1}+\ldots+1$ points, then each subsequent hyperplane meets the "first" hyperplane in $q^{n-2}+\ldots+1$ points, and thus adds at most $q^{n-1}$ new points to the union. But this bound clearly cannot be met without violating the condition that the intersection of all hyperplanes be trivial.

In the plane ($n=2$), the actual lower bound on a blocking set is $q+\sqrt{q}+1$, due to Bruen [Bruen, A., Blocking sets finite projective planes, SIAM J. Appl. Math. 21 (1971), 380–892]. The lower bound is met if and only if the blocking set is a Baer subplane (i.e., a subplane of order $\sqrt{q}$), which occurs only if $q$ is square. There is a substantial literature regarding the situation when $q$ is not square; Hirschfeld [Hirschfeld, J W P, 'Blocking sets', Projective Geometries over Finite Fields (Oxford, 1998; online edn, Oxford Academic, 31 Oct. 2023), https://doi.org/10.1093/oso/9780198502951.003.0013, accessed 10 Nov. 2024.] provides an entire chapter surveying results in this direction.

The situation in higher dimensions is not nearly as well studied. A good reference which addresses some of the issues is Sziklai [P. Sziklai. On small blocking sets and their linearity. J. Combin. Theory Ser. A, 115:1167–1182, 2008.]. In particular, this article contains some relatively small examples of blocking sets which are based on subgeometries, but I do not believe there is any sort of general proof of a lower bound here.

$\endgroup$
1
$\begingroup$

In general, $m\geq n+q-1$ and this is tight, see Lemma 2.3 and Remark 2.4 in Melissa Lee, Tomasz Popiel, Gabriel Verret, Derangements in permutation groups with two orbits, https://arxiv.org/pdf/2506.11396

$\endgroup$

You must log in to answer this question.

Start asking to get answers

Find the answer to your question by asking.

Ask question

Explore related questions

See similar questions with these tags.